Difference between revisions of "2006 AMC 12B Problems/Problem 24"

(Problem)
Line 2: Line 2:
  
 
== Problem ==
 
== Problem ==
{{problem}}
+
Let <math>S</math> be the set of all point <math>(x,y)</math> in the coordinate plane such that <math>0 \le x \le \frac{\pi}{2}</math> and <math>0 \le y \le \frac{\pi}{2}</math>.  What is the area of the subset of <math>S</math> for which
 +
 
 +
<cmath>
 +
\sin^2x-\sin x \sin y + \sin^2y \le \frac34?
 +
</cmath>
 +
 
 +
<math>
 +
\mathrm{(A)}\ \dfrac{\pi^2}{9}
 +
\qquad
 +
\mathrm{(B)}\ \dfrac{\pi^2}{8}
 +
\qquad
 +
\mathrm{(C)}\ \dfrac{\pi^2}{6}
 +
\qquad
 +
\mathrm{(D)}\ \dfrac{3\pi^2}{16}
 +
\qquad
 +
\mathrm{(E)}\ \dfrac{2\pi^2}{9}
 +
</math>
  
 
== Solution ==
 
== Solution ==

Revision as of 22:09, 16 April 2009

This is an empty template page which needs to be filled. You can help us out by finding the needed content and editing it in. Thanks.

Problem

Let $S$ be the set of all point $(x,y)$ in the coordinate plane such that $0 \le x \le \frac{\pi}{2}$ and $0 \le y \le \frac{\pi}{2}$. What is the area of the subset of $S$ for which

\[\sin^2x-\sin x \sin y + \sin^2y \le \frac34?\]

$\mathrm{(A)}\ \dfrac{\pi^2}{9} \qquad \mathrm{(B)}\ \dfrac{\pi^2}{8} \qquad \mathrm{(C)}\ \dfrac{\pi^2}{6} \qquad \mathrm{(D)}\ \dfrac{3\pi^2}{16} \qquad \mathrm{(E)}\ \dfrac{2\pi^2}{9}$

Solution

See also

2006 AMC 12B (ProblemsAnswer KeyResources)
Preceded by
Problem 23
Followed by
Problem 25
1 2 3 4 5 6 7 8 9 10 11 12 13 14 15 16 17 18 19 20 21 22 23 24 25
All AMC 12 Problems and Solutions